4.12 Exemple de test portant sur la matière de ce chapitre

  1. Trouvez une matrice \(\mathbf{A}\) telle que \[\begin{bmatrix} u + v - w \\ 2u + w\end{bmatrix} = \mathbf{A} \begin{bmatrix} u \\ v \\ w\end{bmatrix}.\]

  2. Calculez le produit \(\begin{bmatrix} 1 & 2 \\ -1 & 1 \\ 1 & 0\end{bmatrix} \begin{bmatrix} 3 & 1 & 2 \\ -1 & 0 & 4\end{bmatrix}\).

  3. Soient \(\mathbf{A} = \left[\begin{array}{ccc} 1 & -1 & 1 \\ 0 & 1 & 3 \\ -2 & 1 & 0\\ \end{array}\right]\) et \(\mathbf{B} = \left[\begin{array}{ccc} 1 & -1 & 1\\ 0 & 3 & 9 \\ 0 & -1 & 2\\ \end{array}\right]\). Trouvez une matrice \(\mathbf{M}\) telle que \(\mathbf{M}\mathbf{A} = \mathbf{B}.\)

  4. Soit \(\mathbf{A} = \begin{bmatrix} 1 & 0 & 2\\ 1 & 3 & 3\\ 0 & 1 & 0\end{bmatrix}.\) Trouvez toutes les solutions des systèmes \(\mathbf{A}\mathbf{x} = \mathbf{e}^{(i)}\) pour \(i = 1,2,3\), où \(\mathbf{e}^{(i)}\) dénote la \(i\)-ième colonne de \(\mathbf{I}_3.\)

  5. Déterminez la matrice inverse de \(\begin{bmatrix} -i & 1 \\ 2 & 0\end{bmatrix} \in \mathbb{C}^{2\times 2}.\)

  6. Soit \(\mathbf{A} = \begin{bmatrix} 1 & 0 & 1 \\ 1 & 1 & 0 \\ 0 & 1 & 0\end{bmatrix}\) définie sur \(GF(2).\) Déterminez \(\mathbf{A}^{-1}.\)

  7. Montrez que \(\mathbf{A} = \begin{bmatrix} 4 & -1 & 2\\ -2 & 1 & 1\\ 2 & 0 & 3\end{bmatrix}\) est singulière.

  8. Simplifiez l’expression suivante : \(\left(\begin{bmatrix} 1 \\ 2 \end{bmatrix} \begin{bmatrix} 1 & -1 \end{bmatrix} - \begin{bmatrix} 2 & -1 \\ 0 & 1\end{bmatrix}^3\right)^\mathsf{T}.\)

  9. Soit \(\mathbf{A} = \begin{bmatrix} a & b \\ c & d\end{bmatrix}\in \mathbb{R}^{2\times 2}\). Démontrez que \(\mathbf{A}\) est inversible si et seulement si \(ad - bc \neq 0\).

  10. Soit \(\mathbb{K}\) un corps. Soient \(\mathbf{A} \in \mathbb{K}^{m\times n}\) et \(\lambda \in \mathbb{K}\). Démontrez que \((\lambda \mathbf{A})^\mathsf{T}= \lambda \mathbf{A}^\mathsf{T}\).

Solutions

  1. Notons d’abord que \(\mathbf{A}\) doit être de taille \(2 \times 3\). La première colonne de \(\mathbf{A}\) doit contenir les coefficients de \(u\), la deuxième colonne les coefficients de \(v,\) et la dernière les coefficients de \(w\). Ainsi, \(\mathbf{A} = \begin{bmatrix} 1 & 1 & -1 \\ 2 & 0 & 1\end{bmatrix}\).

  2. Notons que \[\begin{align*} & \begin{bmatrix} 1 & 2 \\ -1 & 1 \\ 1 & 0\end{bmatrix} \begin{bmatrix} 3 & 1 & 2 \\ -1 & 0 & 4\end{bmatrix} \\ = & \begin{bmatrix} 1\cdot 3 + 2 \cdot (-1) & 1 \cdot 1 + 2\cdot 0 & 1\cdot 2 + 2\cdot 4 \\ (-1)\cdot 3 + 1 \cdot (-1) & (-1) \cdot 1 + 1\cdot 0 & (-1)\cdot 2 + 1\cdot 4 \\ 1\cdot 3 + 0 \cdot (-1) & 1 \cdot 1 + 0\cdot 0 & 1\cdot 2 + 0\cdot 4 \end{bmatrix} \\ = & \begin{bmatrix} 1 & 1 & 10 \\ -4 & -1 & 2 \\ 3 & 1 & 2 \end{bmatrix} \end{align*}\]

  3. En appliquant \(L_2 \leftarrow 3 L_2\) à \(\mathbf{A}\), on obtient \(\begin{bmatrix} 1 & -1 & 1 \\ 0 & 3 & 9 \\ -2 & 1 & 0 \end{bmatrix}.\) En appliquant \(L_3 \leftarrow L_3 + 2 L_1\), on obtient \(\begin{bmatrix} 1 & -1 & 1 \\ 0 & 3 & 9 \\ 0 & -1 & 2 \end{bmatrix}.\) Cette matrice est précisément \(\mathbf{B}\).

    Les matrices élémentaires correspondantes à la première opération et à la deuxième opération sont, respectivement, \(\begin{bmatrix} 1 & 0 & 0 \\ 0 & 3 & 0 \\ 0 & 0 & 1\\ \end{bmatrix}\) et \(\begin{bmatrix} 1 & 0 & 0 \\ 0 & 1 & 0 \\ 2 & 0 & 1\\ \end{bmatrix}\).

    Ainsi, \[\begin{align*} \mathbf{M} & = \begin{bmatrix} 1 & 0 & 0 \\ 0 & 1 & 0 \\ 2 & 0 & 1 \end{bmatrix} \begin{bmatrix} 1 & 0 & 0 \\ 0 & 3 & 0 \\ 0 & 0 & 1 \end{bmatrix} \\ & = \begin{bmatrix} 1 & 0 & 0 \\ 0 & 3 & 0 \\ 2 & 0 & 1 \end{bmatrix}, \end{align*}\] et on observe aisément que \(\mathbf{M} \mathbf{A} = \mathbf{B}\).

    Remarque. Observons que l’on peut obtenir \(\mathbf{M}\) en appliquant la même suite d’opérations élémentaires sur les lignes de \(\mathbf{I}_3,\) en raison du fait que chaque multiplication (à gauche) par une matrice élémentaire a la même effect que l’opération élémentaire sur les lignes associée avec cette matrice. Notons que cela réduit grandement les calculs lorsque le nombre d’opérations élémentaires sur les lignes est élevé.

  4. La matrice augmentée généralisée capturant les trois systèmes est \[\left[\begin{array}{ccc|ccc} 1 & 0 & 2 & 1 & 0 & 0 \\ 1 & 3 & 3 & 0 & 1 & 0 \\ 0 & 1 & 0 & 0 & 0 & 1 \end{array}\right].\] L’application de \(L_2 \leftarrow L_2 - L_1\) donne \[\left[\begin{array}{ccc|ccc} 1 & 0 & 2 & 1 & 0 & 0 \\ 0 & 3 & 1 & -1 & 1 & 0 \\ 0 & 1 & 0 & 0 & 0 & 1 \end{array}\right].\]

    L’application de \(L_2 \leftarrow L_2 - 3L_3\) donne \[\left[\begin{array}{ccc|ccc} 1 & 0 & 2 & 1 & 0 & 0 \\ 0 & 0 & 1 & -1 & 1 & -3 \\ 0 & 1 & 0 & 0 & 0 & 1 \end{array}\right].\]

    L’application de \(L_1 \leftarrow L_1 - 2L_2\) donne \[\left[\begin{array}{ccc|ccc} 1 & 0 & 0 & 3 & -2 & 6 \\ 0 & 0 & 1 & -1 & 1 & -3 \\ 0 & 1 & 0 & 0 & 0 & 1 \end{array}\right].\] L’application de \(L_2 \leftrightarrow L_3\) donne \[\left[\begin{array}{ccc|ccc} 1 & 0 & 0 & 3 & -2 & 6 \\ 0 & 1 & 0 & 0 & 0 & 1 \\ 0 & 0 & 1 & -1 & 1 & -3 \end{array}\right].\]

    Ainsi, \(\begin{bmatrix} x_1 \\x_2\\ x_3\end{bmatrix} = \begin{bmatrix} 3 \\ 0 \\ -1 \end{bmatrix}\) est l’unique solution de \(\mathbf{A}\mathbf{x} = \mathbf{e}^{(1)}\), \(\begin{bmatrix} x_1 \\x_2\\ x_3\end{bmatrix} = \begin{bmatrix} -2 \\ 0 \\ 1 \end{bmatrix}\) est l’unique solution de \(\mathbf{A}\mathbf{x} = \mathbf{e}^{(2)}\), et \(\begin{bmatrix} x_1 \\x_2\\ x_3\end{bmatrix} = \begin{bmatrix} 6 \\ 1 \\ -3 \end{bmatrix}\) est l’unique solution de \(\mathbf{A}\mathbf{x} = \mathbf{e}^{(3)}\).

  5. Nous appliquons la réduction à \(\left[\begin{array}{cc|cc} -i & 1 & 1 & 0 \\ 2 & 0 & 0 & 1 \end{array}\right],\) afin d’obtenir \[\begin{align*} & \left[\begin{array}{cc|cc} -i & 1 & 1 & 0 \\ 2 & 0 & 0 & 1 \end{array}\right] \\ \xrightarrow{L_1\leftarrow i L_1}~ & \left[\begin{array}{cc|cc} 1 & i & i & 0 \\ 2 & 0 & 0 & 1 \end{array}\right] \\ \xrightarrow{L_2\leftarrow L_2 - 2 L_1}~ & \left[\begin{array}{cc|cc} 1 & i & i & 0 \\ 0 & -2i & -2i & 1 \end{array}\right] \\ \xrightarrow{L_2\leftarrow \frac{i}{2} L_1}~ & \left[\begin{array}{cc|cc} 1 & i & i & 0 \\ 0 & 1 & 1 & \frac{i}{2} \end{array}\right] \\ \xrightarrow{L_1\leftarrow L_1 - i L_2} ~& \left[\begin{array}{cc|cc} 1 & 0 & 0 & \frac{1}{2} \\ 0 & 1 & 1 & \frac{i}{2} \end{array}\right] \end{align*}\] La matrice inverse est \(\begin{bmatrix} 0 & \frac{1}{2} \\ 1 & \frac{i}{2} \end{bmatrix}\).

  6. Nous appliquons la réduction à \(\left[\begin{array}{ccc|ccc} 1 & 0 & 1 & 1 & 0 & 0 \\ 1 & 1 & 0 & 0 & 1 & 0 \\ 0 & 1 & 0 & 0 & 0 & 1 \end{array}\right],\) afin d’obtenir \[\begin{align*} & \left[\begin{array}{ccc|ccc} 1 & 0 & 1 & 1 & 0 & 0 \\ 1 & 1 & 0 & 0 & 1 & 0 \\ 0 & 1 & 0 & 0 & 0 & 1 \end{array}\right] \\ \xrightarrow{L_2 \leftarrow L_2 + L_1}~ & \left[\begin{array}{ccc|ccc} 1 & 0 & 1 & 1 & 0 & 0 \\ 0 & 1 & 1 & 1 & 1 & 0 \\ 0 & 1 & 0 & 0 & 0 & 1 \end{array}\right] \\ \xrightarrow{L_2 \leftrightarrow L_3}~ & \left[\begin{array}{ccc|ccc} 1 & 0 & 1 & 1 & 0 & 0 \\ 0 & 1 & 0 & 0 & 0 & 1 \\ 0 & 1 & 1 & 1 & 1 & 0 \end{array}\right] \\ \xrightarrow{L_3 \leftarrow L_3 + L_2}~ & \left[\begin{array}{ccc|ccc} 1 & 0 & 1 & 1 & 0 & 0 \\ 0 & 1 & 0 & 0 & 0 & 1 \\ 0 & 0 & 1 & 1 & 1 & 1 \end{array}\right] \\ \xrightarrow{L_1 \leftarrow L_1 + L_3}~ & \left[\begin{array}{ccc|ccc} 1 & 0 & 0 & 0 & 1 & 1 \\ 0 & 1 & 0 & 0 & 0 & 1 \\ 0 & 0 & 1 & 1 & 1 & 1 \end{array}\right] \end{align*}\] Donc, \(\mathbf{A}^{-1} = \begin{bmatrix} 0 & 1 & 1 \\ 0 & 0 & 1\\ 1 & 1 & 1 \end{bmatrix}\).

  7. Nous devons trouver un 3-uplet \(\mathbf{x}\) tel que \(\mathbf{A}\mathbf{x} = \mathbf{0}\). Pour ce faire, il suffit de réduire \(\mathbf{A}\) : \[\begin{align*} & \begin{bmatrix} 4 & -1 & 2\\ -2 & 1 & 1\\ 2 & 0 & 3\end{bmatrix} \\ \xrightarrow{L_3 \leftarrow L_3 + L_2}~ & \begin{bmatrix} 4 & -1 & 2\\ -2 & 1 & 1\\ 0 & 1 & 4\end{bmatrix} \\ \xrightarrow{L_2 \leftarrow L_2 + \frac{1}{2}L_1}~& \begin{bmatrix} 4 & -1 & 2\\ 0 & \frac{1}{2} & 2\\ 0 & 1 & 4 \end{bmatrix} \\ \xrightarrow{L_2 \leftarrow 2L_2}~& \begin{bmatrix} 4 & -1 & 2\\ 0 & 1 & 4\\ 0 & 1 & 4\end{bmatrix} \\ \xrightarrow{L_3 \leftarrow L_3 - L_2}~& \begin{bmatrix} 4 & -1 & 2\\ 0 & 1 & 4\\ 0 & 0 & 0\end{bmatrix} \\ \xrightarrow{L_1 \leftarrow L_1 + L_2}~& \begin{bmatrix} 4 & 0 & 6\\ 0 & 1 & 4\\ 0 & 0 & 0\end{bmatrix} \\ \xrightarrow{L_1 \leftarrow \frac{1}{4}L_1}~& \begin{bmatrix} 1 & 0 & \frac{3}{2}\\ 0 & 1 & 4\\ 0 & 0 & 0 \end{bmatrix} \\ \end{align*}\] Ainsi, \(\mathbf{x} = \begin{bmatrix} -\frac{3}{2} \\ -4 \\1 \end{bmatrix}\) satisfait à \(\mathbf{A}\mathbf{x} = \mathbf{0}\), ce qui implique que \(\mathbf{A}\) est singulière puisque \(\mathbf{x}\) est une solution non triviale.

  8. Notons que \[\begin{align*} & \left(\begin{bmatrix} 1 \\ 2 \end{bmatrix} \begin{bmatrix} 1 & -1 \end{bmatrix} - \begin{bmatrix} 2 & -1 \\ 0 & 1\end{bmatrix}^3\right)^\mathsf{T}\\ = \; & \left(\begin{bmatrix} 1 & -1 \\ 2 & -2 \end{bmatrix} - \begin{bmatrix} 2 & -1 \\ 0 & 1\end{bmatrix}^3\right)^\mathsf{T}\\ = \; & \left(\begin{bmatrix} 1 & -1 \\ 2 & -2 \end{bmatrix} - \begin{bmatrix} 2 & -1 \\ 0 & 1\end{bmatrix} \begin{bmatrix} 2 & -1 \\ 0 & 1\end{bmatrix}^2\right)^\mathsf{T}\\ = \; & \left(\begin{bmatrix} 1 & -1 \\ 2 & -2 \end{bmatrix} - \begin{bmatrix} 2 & -1 \\ 0 & 1\end{bmatrix} \left(\begin{bmatrix} 2 & -1 \\ 0 & 1\end{bmatrix} \begin{bmatrix} 2 & -1 \\ 0 & 1\end{bmatrix}\right)\right)^\mathsf{T}\\ = \; & \left(\begin{bmatrix} 1 & -1 \\ 2 & -2 \end{bmatrix} - \begin{bmatrix} 2 & -1 \\ 0 & 1\end{bmatrix} \begin{bmatrix} 4 & -3 \\ 0 & 1\end{bmatrix}\right)^\mathsf{T}\\ = \; & \left(\begin{bmatrix} 1 & -1 \\ 2 & -2 \end{bmatrix} - \begin{bmatrix} 8 & -7 \\ 0 & 1\end{bmatrix}\right)^\mathsf{T}\\ = \; & \begin{bmatrix} -7 & 6 \\ 2 & -3 \end{bmatrix}^\mathsf{T} = \begin{bmatrix} -7 & 2 \\ 6 & -3 \end{bmatrix} \end{align*}\]

  9. Montrons d’abord que si \(ad - bc = 0\), alors \(\mathbf{A}\) est singulière et donc non inversible.

    Si \(a = c = 0\), alors \(\mathbf{A}\) est évidemment singulière puisque \(\mathbf{A}\begin{bmatrix} 1 \\ 0\end{bmatrix} = \begin{bmatrix} 0 \\ 0\end{bmatrix}\).

    Supposons d’abord que \(a\) et \(c\) ne sont pas tous deux nuls. Alors au moins un uplet parmi \(\mathbf{u} = \begin{bmatrix} d \\ -c\end{bmatrix}\) et \(\mathbf{v} = \begin{bmatrix} b \\ -a \end{bmatrix}\) n’est pas égale à \(\begin{bmatrix} 0 \\0\end{bmatrix}\). Mais \(\mathbf{A}\mathbf{u} = \begin{bmatrix} ad - bc \\ cd - dc\end{bmatrix} = \begin{bmatrix} 0 \\0\end{bmatrix}\) et \(\mathbf{A}\mathbf{v} = \begin{bmatrix} ab - ba \\ cb - da\end{bmatrix} = \begin{bmatrix} 0 \\0\end{bmatrix}\). Ainsi, \(\mathbf{A}\) est singulière et non inversible.

    Montrons maintenant que si \(ad - bc \neq 0\), alors \(\mathbf{A}\) est inversible. Supposons que \(a \neq 0\). Alors la réduction suivante est valide : \[\begin{align*} & \left[\begin{array}{cc|cc} a & b & 1 & 0 \\ c & d & 0 & 1 \\ \end{array}\right] \\ \xrightarrow{L_1 \leftarrow \frac{1}{a} L_1}~ & \left[\begin{array}{cc|cc} 1 & \frac{b}{a} & \frac{1}{a} & 0 \\ c & d & 0 & 1 \end{array}\right] \\ \xrightarrow{L_2 \leftarrow L_2 -c L_1}~ & \left[\begin{array}{rc|cc} 1 & \frac{b}{a} & \frac{1}{a} & 0 \\ 0 & \frac{ad-bc}{a} & -\frac{c}{a} & 1 \end{array}\right] \\ \xrightarrow{L_2 \leftarrow \frac{a}{ad-bc}L_2 }~ & \left[\begin{array}{rc|cc} 1 & \frac{b}{a} & \frac{1}{a} & 0 \\ 0 & 1 & -\frac{c}{ad-bc} & \frac{a}{ad-bc} \end{array}\right] \end{align*}\]

    Ainsi, \(\mathbf{A}\) est inversible.

    Supposons maintenant que \(a = 0\). Puisque \(ad -bc \neq 0\), on doit avoir \(c \neq 0\). Les détails sont semblables au cas précédant et sont laissés en exercice.

    Remarque. La solution présentée ici est longue et non intuitive. Nous verrons plus tard que l’on obtient une démonstration en utilisant le déterminant.

  10. Soit \(\mathbf{B} = \lambda \mathbf{A}\). Montrons que \(\mathbf{B}^\mathsf{T}= \lambda \mathbf{A}^\mathsf{T}\).

    Notons que l’élément \((i,j)\) de \(\mathbf{B}\) est \(\lambda a_{ij}\). Ainsi, l’élément \((i,j)\) de \(\mathbf{B}^\mathsf{T}\) est l’élément \((j,i)\) de \(\mathbf{B}\) et est donc \(\lambda a_{ji}\).

    Mais l’élément \((i,j)\) de \(\lambda \mathbf{A}^\mathsf{T}\) est lui-même le \(\lambda\)-multiple de l’élément \((i,j)\) de \(\mathbf{A}^\mathsf{T}\), qui se retrouve également à être \(\lambda a_{ji}\). Ainsi, \(\mathbf{B}^\mathsf{T}= \lambda \mathbf{A}^\mathsf{T}\).